ChaseDream
搜索
返回列表 发新帖
查看: 2545|回复: 3

逻辑题题目问法大总结

[复制链接]
发表于 2008-9-30 23:48:00 | 显示全部楼层

逻辑题题目问法大总结

逻辑题题目问法大总结

The most important key to Critical reasoning is whether you can reach the point for what the questions ask. I itemize all the questions as follows.
! k9 }3 v* a) b( t4 `* {2 Y: e
加强题:
+ \1 w2 b8 q- [3 v1.Which one of the following statements, if added as a premise to the argument, would make the conclusion valid / strengthen the argument?
/ h! V6 q+ S6 O2.Which one of the following, if true, would make the conclusion in the passage a logical conclusion?
1 |$ j2 g. p, h$ e9 ^4 e0 F3.Which one of the following is an additional piece of information that would support the conclusion?' E7 y0 ]: e; B+ r- ^5 z
4.A would be most likely to contribute to an explanation of fact 1 if which of the following were also true?. x  {5 h4 Y$ [4 J/ J# V: M
5.Which of the following, if true, could proponents of the plan above most appropriately cite as a piece of evidence for the soundness of their plan? According to this question, you should focus on the effective of plan and then find out which of the following would strengthen the effect. 0 o2 M% f# l* }( b  I5 N* I
6.Someone’s argument as it is presented in the passage above would be most strengthened if which of the following were true?
定位+加强
$ A3 l, r0 W: b( A$ s( v$ @5 e. o7.
$ E) a: `3 X- x* B+ G) |
削弱题2 G; J+ N2 I2 B5 t5 `
1.Which one of the following statements, if true, most clearly undermines / seriously weaken the conclusion / argument?
+ M* p2 G9 S. U  p+ a2.Which one of the following assertions, if true, provides the most effective challenge to the author's conclusion?" b7 E' A* z. F! i; Y# ]7 S+ R/ n
3.Which of the following, if true, would cast the most serious doubt on the accuracy of the group’s contention.
定位+削弱
5 y6 }1 ]9 b. A# A* P: r1 N4.Which of the following, assuming that it is a realistic possibility (=if true), argues most strongly against the suggestion above.
& j8 V: l, t2 l5.Which of the following statements about something, if true, would strongly suggest that the suggestion or the plan is flawed? Suggesting something is wrong is meaning weakening.
( |; s7 m0 [5 k+ a+ t8 l) o8 w6.Which of the following statements, if true, is the best basis for a criticism of historical costing as an economically sound pricing method for military contracts?
可以理解为对the soundness of “historical costing”的削弱,也可以理解为对a criticism of historical costing的加强。
3 V" ~( ~$ A! F; r7.Which of the following, if it were discovered, would be pertinent evidence against the speculation above? Pertinent evidence against something clearly demonstrates the effective weakening.
& W( i4 t# ^) ^# O  [/ I9 R3 q8.Which of the following statements, if true, provides the best evidence that someone’s reasoning is flawed?
! `! @$ b2 K3 R7 t7 @# {3 t( A9.The use of something could not be effective in doing something if which of the following were true?
, _/ Y) N( @+ ^: A9 o4 Y5 G10.Which of the following, if known, is evidence that contradicts the hypothesis stated above?
# Q# b* q1 S& N: V1 l11.Which of the following, if true, could present the most serious disadvantage for AAA in doing something?
) t* q3 R8 x% d0 p4 E% b12./ ~4 \7 u: A5 I# O* U
假设题. {' _$ p" X( [  |2 y
1.Which one of the following is an assumption on which the author's argument relies?
  ^( ~! j* k6 o5 f2.Which one of the following most accurately states a hidden assumption that the author must make in order to advance the argument above?: ?0 `7 T& R% K0 ?/ T7 g
3.Which one of the following is a presupposition essential to the reasoning in the passage above?
* V% _, A: ~( R1 `* z+ c4.The conclusion drawn in the first sentence depends on which of the following assumptions?
6 m# j; N9 k4 N$ G/ M3 q- p0 y5.
! G% R- i. `( ^6 i  I9 q! o
评价题) s6 E  V. p5 T% u) r4 X
1.Which one of the following is the most accurate evaluation of the author's reply?
; i4 w4 K! `( `) W, g2.Which one of the following principles, if valid, justifies the author's argument?* P8 D# V6 K9 O' J2 {
3.Which one of the following would it be most helpful to know in order to evaluate the argument?
6 Q# G: j1 D+ V# y4 \/ `% a. O5 p4.The answer to which of the following questions is least directly relevant to the union leader’s consideration of whether attempting a boycott of Gasco will lead to acceptance of their contract proposal?3 i' \( S  R/ v; {9 S
5.Which of the following investigation is most likely to yield significant information that would help to evaluate the researcher’s hypothesis?
! u; e. p: {5 Q3 l6.# V$ w' }' k; g( b9 p$ I- |
解释题
6 A, o8 r/ E& ~- b% y) m8 `1.The author concludes that ... partly because...# Q8 S) [$ a6 p
2.Which one of the following, if true, argues most strongly against the explanation reported in the passage?3 x" P* g  _9 _% n
3.Which one of the following, if true, most helps to explain why something happens?
1 P+ E6 H6 s; l& |% y4.Which one of the following principles, if established, would justify the judgments about A's & B's actions?  [7 L8 a/ Z6 t% Z! _! R0 }
矛盾解决题(也可归类到解释题里)2 H+ m; A9 ]. P* c
1.Which one of the following, if true, most helps to resolve the apparent discrepancy between the two?- {. n8 ]$ G; F! X
2.Which one of the following, if true, most helps explain the difference above?
- ^6 U  ~4 j' H7 N' Q, Z! V3.Which one of the following, if true, most helps to reconcile the author's decision with the goal stated in the passage?
0 @$ t  B$ a( {- |( r5 \) w$ ]4.
# M* S. t: ?" W. p8 {& c- z
结论(归纳)题
2 E# i4 A3 f% t8 _2 C/ K- d1.It can be concluded from the statements above that...' p3 P! r0 `+ L
2.Which one of the following statements most accurately expresses the author's main point?& g7 ]0 R( ^. E% J: \
3.Which one of the following statements CANNOT be true?
6 Y. q; {: y  H- f, s7 I; d! L4.Which one of the following inferences can be most reliably drawn from the passage above?: ~( B8 N# T% H* ^' L9 y1 W3 `
5.The passage as a whole is structured to lead to which one of the following conclusions?5 }5 S6 E4 {8 S$ r  C( O
6.Which one of the following is the best statement of the primary point of the passage?- f$ i3 J1 K- N# M6 A5 e
7.Each of the following can be inferred from his argument EXCEPT...
; }4 q1 M) J6 y) c, L5 \. _8.Which one of the following claims is central to the author's argument?. T3 H2 B( x+ X' j( b+ D  R' I" p
9.If the statements above are true, then which one of the following must also be true?
' e( M, _) o3 L2 P6 t0 f/ M10.! s4 m! b3 ]) B, }2 O, S2 X  ]# ]
信息支持(反向加强--应该也可以算是归纳题)题5 R' p0 b! L& ^) S
11.1.Which one of the following is most strongly supported by the information above?
1 K6 X& a& @- F, s8 t12.The statement above, if true, best support which one of the following claim/ conclusions/which of the following assertions?. j% Z; L8 ]' j8 \  J
13.5 y* c4 P+ q( Z/ q! }0 u  J

7 x/ ], B6 P" L7 ~* t
句子完成(划线填空)题其实最好分别,就是一看到有划线的就可以判断是划线题。
6 q# [0 y, M" C, ~2 ^1.Which one of the following is the best completion of the argument above?; Based on the passage above, which phrase does NOT provide a logical completion to the following sentence?
9 s7 J( b" ~0 c% s  [9 I2 s2.
/ u% j/ K" y& [6 t, y! Y' c' ]: G; ~: P
推理缺陷题! b+ ^: w0 N: _( B0 ]- N3 O+ h
1.What is the flaw in the reasoning above?
$ F8 Z  ?( i2 S/ E) L2.The author's argument is logically flawed in that it...+ ]: @( m. i8 w+ h
3.The source of the man's erroneous reasoning is his.../ D$ e; w( @3 P2 Y9 @4 F
4.The reasoning in the argument is most vulnerable to the criticism that it...
* [( f3 P4 u/ P5.Which of the following indicates a flaw in the reasoning above?+ C# o2 c8 o2 @- _0 t
6.Which of the following challenges indicates the most serious weakness in the attempted explanation described above?9 ~4 y  O3 t& a9 t5 e9 G
7.
, T" h' v; s/ u0 C" g) C" B2 \
逻辑相似性题1 g+ L8 C( H* k4 P8 N
1.Which one of the following is most similar in logic to the argument above?
5 L5 Z& g( ~. C3 X" H  g" H2.Which one of the following exhibits the same logical flaw as that exhibited above?
! x2 K  L! @1 D# K" `; K8 o7 ]* u: S3.Which one of the following could be best supported by the same type of reasoning as that exhibited in the passage?* k; W) @  m0 f1 P+ T
4.Which one of the following contains the error of reasoning described by the author in the passage?; Which one of the following most closely parallels the flawed pattern of reasoning in the author's reply?
2 @% N! [2 [8 p5.
* `$ B! W$ i# N- w# C5 U% r' p
逻辑相悖题
6 U; X& L7 a, _8 @1.In which one of the following situations is the principle expressed most clearly violated?
# r3 Q, g6 F( I% f: p9 [0 p8 V: x2.. n9 C( K0 ~8 Q1 D& T2 ~' Z
两人争论内容题
8 v) W1 \: F$ d1.The source of A & B's dispute is their lack of agreement on which one of the following terms?! r( B1 l# B2 X; L* n, W$ Y
2.Which one of the following is the point at issue between A & B?
, H8 i' c0 G8 K5 ^: n3.9 m- _: w1 }1 J7 P9 z
逻辑策略方法题
( F8 R# ?: B& V" R# V1.Which one of the following is the best line of causal analysis for A to do?
) r. ~, t2 d6 O0 f. d2 b# w2.The author establishes his position in the passage by doing which one of the following?
% t" ~4 J# \; B3 l1 ?$ K1 {1 S+ i5 v3.The author of the passage argues by...0 t3 Q3 d) G9 j; |
4.The argument seeks to do which one of the following?9 \' k# ?: Y7 H( h3 o1 }4 y6 o
5.A most seriously weakens B's argument by doing which one of the following?
: b  R! g$ ]3 U5 \6 y4 c6.Which one of the following techniques of argument does the author use in the passage?; A's statement most closely conforms to which one of the following principles?; The relationship if A's response to B's argument is that A's response...; A responds to B's argument by...How is A's response related to B's argument?: Y; m6 o8 R3 O+ j# Z! ]' o
句子作用题(bold face
6 [; ^# A  Y) N; |+ d1.In the passage above, the two portions in boldface play which of the following roles?

 

发表于 2014-10-18 09:41:19 | 显示全部楼层
赞!!!!!!!!!!!!!
发表于 2019-5-21 18:01:49 | 显示全部楼层
感谢分享!!!
发表于 2019-5-22 23:06:21 | 显示全部楼层
感谢分享!               
您需要登录后才可以回帖 登录 | 立即注册

Mark一下! 看一下! 顶楼主! 感谢分享! 快速回复:

手机版|ChaseDream|GMT+8, 2024-4-18 18:23
京公网安备11010202008513号 京ICP证101109号 京ICP备12012021号

ChaseDream 论坛

© 2003-2023 ChaseDream.com. All Rights Reserved.

返回顶部